LSAT and Law School Admissions Forum

Get expert LSAT preparation and law school admissions advice from PowerScore Test Preparation.

 Administrator
PowerScore Staff
  • PowerScore Staff
  • Posts: 8916
  • Joined: Feb 02, 2011
|
#91286
Complete Question Explanation

The correct answer choice is (B).

Answer choice (A):

Answer choice (B): This is the correct answer choice.

Answer choice (C):

Answer choice (D):

Answer choice (E):

This explanation is still in progress. Please post any questions below!
User avatar
 dafastlane
  • Posts: 1
  • Joined: Sep 27, 2022
|
#97504
Hi there - I would love some advice for how to complete this question quicker. Is there a particular strategy I should use?
 Rachael Wilkenfeld
PowerScore Staff
  • PowerScore Staff
  • Posts: 1358
  • Joined: Dec 15, 2011
|
#97533
Hi dafastlane,

The strategy I took was focusing on the first position. The first position cannot be K, F or J, so it must be only M H or L. For each answer choice, I looked first at how each answer choice would impact that first position. I should also note that as part of my set up, I had a conditional that said if H is 1, what else would happen. If H is 1, L is 4, K would be 5, and F would be 6. That means that M has to be 2, and J has to be 3. So if H is 1, we have the whole order (HMJLKF). I use that frequently to eliminate answer choices.

For answer choice (A), F immediately before M means that we have a FMJ block. That takes M out of contention for slot 1, as well as H (because F is last when H is 1). But we still could have L in slot 1 without an issue. This is one that I would consider unlikely.

For answer choice (B) if H is immediately before L, we have an HL block and an MJ block. H then cannot be 1 (because L would be 4). L can't be first because H would be right before it. That means that M would have to be first. We'd do MJHLKF. But when F is 6, J has to be 3. So none of the possible options for slot 1 work. This is the one that must be false.

For answer choice (C) if J is immediately before F, we still have M and L as possible options for space 1.

For answer choice (D) if K is immediately before M, we can't use H as 1 (because L is before K in that instance) and we can't use M as 1 (since K is before it here). But L could still be 1. It still looks possible.

For answer choice (E) if L is before H, L could still be first. It looks like a possibility.

So the only answer with a violation was answer choice (B). I still read all 5 just in case, but I feel confident that I found a violation in answer choice (B).

Hope that helps!

Get the most out of your LSAT Prep Plus subscription.

Analyze and track your performance with our Testing and Analytics Package.